Question

In: Physics

A rectangular coil with resistance R has N turns, each of length and width w as...

A rectangular coil with resistance R has N turns, each of length scripted l and width w as shown in Figure P31.29. The coil moves into a uniform magnetic field vector B with constant velocity vector v . What are the magnitude and direction of the total magnetic force on the coil for the following situations? (Use the following as necessary: N,B, w, v, and R.)

p23-22.gif
Figure P31.29

(a) The coil enters the magnetic field.
F =   [Direction?]
(b) The coil moves within the field.
F =   [Direction?]
(c) The coil leaves the field.
F = [Direction?]

Solutions

Expert Solution

Concepts and reason

The concepts required to solve the given question are magnetic force, Ohm’s law and the relation between the magnetic flux and the induced emf.

First, find an expression for the magnetic force due to a current carrying wire when the coil enters the magnetic field. Then, use Ohm’s law and the Faraday’s law and express the force in terms of the given quantities. In the next step, find the direction of the magnetic force using the right hand rule.

Next, find an expression for the magnetic force when the coil moves within the field from the relation between the magnetic force, current, and the magnetic flux.

After that, find the direction of the magnetic force when the coil is completely in the magnetic field.

In the next part, find the expression for the magnetic force when the coil leaves the field. Finally, find the direction of the magnetic force by applying the right hand rule.

Fundamentals

The force that arises due to the motion of charged particle within the magnetic field is known as the magnetic force.

The magnetic force due to a current carrying wire is given by,

Here, is the magnetic force, is the magnetic field, is the current, is the length of the wire and is the angle between the velocity vector and the magnetic field vector.

In this case, the velocity vector and the magnetic field vector are perpendicular to each other. Hence the angle is.

Thus the magnetic force expression becomes,

For number of turns, the magnetic force is,

According to Faraday’s law of electromagnetic induction is “the rate of change of flux linkage is equal to the induced emf”.

Here, is the magnitude of the induced emf, is the number of turns, and is the rate of change of magnetic flux.

Ohm’s law gives the relation between current, voltage and resistance. The law states that the “electric current is directly proportional to the voltage and inversely proportional to the resistance of the circuit”.

The expression for Ohm’s law is given by,

Here, V is the voltage, I is the current and R is the resistance of the circuit.

(a.1)

Consider the expression for the magnetic force due to a current carrying coil,

…… (1)

According to the Faraday’s law, the induced emf in the circuit is,

…… (2)

The magnetic flux induced in the coil is,

Here, is the magnetic flux, is the number of turns, is the area of the coil, is the angle between the magnetic field and the area vector and is the magnetic field.

In this case, the magnetic field and the area vector are parallel to each other. Hence the angle becomes.

Thus the magnetic flux is,

The area of the rectangular loop is,

Here, is the length and is the width of the loop.

Use for in terms of .

…… (3)

Substitute the equation (3) in equation (2).

Using the relation for the velocity of the coil moves in a magnetic field,

…… (4)

Here, v is the velocity of the coil, is the change in length, and is the time taken.

Substitute equation (4) in the above equation,

…… (5)

The expression for the current flows in the circuit is given by,

Substitute equation (5) in the expression for the current in the circuit.

Substitute the expression for the current in equation (1).

(a.2)

Right hand rule states that, place the right hand thumb in the direction of the magnetic field and the curled fingers gives the direction of current.

In this case, there is an induced emf due to the change in magnetic flux. Thus a magnetic field is generated in the opposite direction of the applied field. It will be pointing out of the page. By applying the right hand rule, the direction of current is counterclockwise.

Now curl the fingers in a direction rotating the current in to the magnetic field. Then thumb gives the direction of the magnetic force.

In this case, the magnetic force is acting towards left.

(b.1)

When the coil moves within the field, the flux linked is given by,

Here, the flux is constant.

Hence there is no induced emf in the coil, so that .

The current in the coil is,

Thus the magnetic force when the coil moves within the field is,

(b.2)

Magnetic flux linked with the coil when it moves within the magnetic field is constant. Hence the change in magnetic flux is zero.

Then, there is no induced emf in the coil also the current in the coil is zero. Thus, the magnetic force is zero. Hence there is no direction for the magnetic force.

(c.1)

As the coil leaves the magnetic field, the rate of change of magnetic flux decreased.

The decreasing rate of change of magnetic flux is,

The induced emf is given by,

Substitute for .

The current flowing through the loop is,

Substitute for .

The magnetic force acting on the coil is,

Substitute for and for .

(c.2)

Here, the rate of change of magnetic flux is decreased. Hence the direction of the induced emf is clockwise. But the current through the portion of the coil in the magnetic field is upwards. Then the magnetic force is acting towards the left.

Ans: Part a.1

The expression for the magnetic force acting on the coil when it enters to the magnetic field is .

Part a.2

The direction of magnetic force when the coil enters to the magnetic field is towards left.

Part b.1

The total magnetic force when the coil moves within the magnetic field is zero.

Part b.2

There is no direction for the magnetic force acting on the coil when it moves within the magnetic field.

Part c.1

The magnetic force acting on the coil when it leaves the magnetic field is .

Part c.2

The direction of magnetic force on the coil when it leaves the magnetic field is towards left.


Related Solutions

A rectangular coil with resistance R has N turns, each of length and width ar as shown in Figure P31.29
A rectangular coil with resistance R has N turns, each of length and width ar as shown in Figure P31.29. The coil moves into a uniform magnetic field B with constant velocity. What are the magnitude and direction of the total magnetic force on the coil (a) as it enters the mag- netic field, (b) as it moves within the field, and (c) as it leaves the field?
A rectangular coil with resistance R has N turns
A rectangular coil with resistance R has N turns, each of length l and width w as shown in Figure P20.25. The coil moves into a uniform magnetic field with constant velocity . What is the magnitude and direction of the total magnetic force on the coil during the following intervals? (Use N, B, w, l,v, and R as needed.)(a) as it enters the magnetic field(b) as it moves within the field(c) as it leaves the field
​The figure below shows a rectangular coil of length 1 and width w consisting of N turns of conducting wire
The figure below shows a rectangular coil of length 1 and width w consisting of N turns of conducting wire, moving to the right with a constant velocity v. The coil moves into a region of uniform magnetic field B, pointing into the page and perpendicular to the plane of the coil. The total resistance of the coil is R. Find the magnitude and direction of the total magnetic force on the coil for the following situations. (Use the following as...
A rectangular coil with N turns and area A is place in a uniform magnetic field...
A rectangular coil with N turns and area A is place in a uniform magnetic field directed into y-direction. (a) What amount of emf is induced in the coil if the coil doesn’t move or rotate? . Let’s say the coil rotates about the z-axis through its center at a constant angular velocity LaTeX: \omegaω. (b) What would be total magnetic flux through the coil as a function of LaTeX: \thetaθ ? (LaTeX: \thetaθ is the angle between the directions...
A circular coil of N turns has radius R, and carries a currenrt I in the counterclockwise direction in each turn.
A circular coil of N turns has radius R, and carries a currenrt I in the counterclockwise direction in each turn. The coil lies in the wy-plane,with its center at the origin. There is a uniform magnetic field B=Bxi + BzK everywhere. Assume both Bx>0 and Bz>0.(a) What is the net force on the coil? Remember that the magnetic field is uniform.(b) Find the magnetic dipole moment μ of the coil.(c) Find the torque vector on the coil.
A high-­viscosity oil is transported through a wide rectangular duct of length L, width W and...
A high-­viscosity oil is transported through a wide rectangular duct of length L, width W and depth 2B via pressure-­driven flow. The duct is inclined at an angle b? below the horizontal plane (gravity may be assumed to act downwards in the vertical direction), and is sufficiently broad that edge effects may be neglected in the transverse (x2) direction. The pressure at the upstream end of the duct (x1=0) is Po, and at the downstream end (x1=L) is PL. The...
Design a rectangular milk carton box of width w, length ll, and height h which holds...
Design a rectangular milk carton box of width w, length ll, and height h which holds 540 cm^3 of milk. The sides of the box cost 1 cent/cm^2 and the top and bottom cost 3 cent/cm^2. Find the dimensions of the box that minimize the total cost of materials used.
A closely wound coil has: radius of 5cm, 200 turns, and Resistance of 10 ohms. There...
A closely wound coil has: radius of 5cm, 200 turns, and Resistance of 10 ohms. There is a 10.0mA of constant current in the search coil. The magnetic field through the coil is 0 at t=0, perpendicular to the area of the coil. The coil is completely inside a uniform magnetic field. (a) Find the magnitude of the magnetic field at t=20.0ms. (b)The magnetic field from part (a) is produced by a larger coil which is outside of the first...
An ideal solenoid, of radius R and n turns per unit length, has a current flowing...
An ideal solenoid, of radius R and n turns per unit length, has a current flowing through it. The current, I, varies with time, t, according to I = I0 + at where I0 and a are constants. A conducting ring of radius, r, is placed inside the solenoid with its axis coinciding with the axis of the solenoid. The ring has a resistance per unit length of H (in units of Ω/m). (a) Use Lenz’s law to determine the...
The rotor in a certain electric motor is a flat, rectangular coil with 89 turns of...
The rotor in a certain electric motor is a flat, rectangular coil with 89 turns of wire and dimensions 2.57 cm by 4.20 cm. The rotor rotates in a uniform magnetic field of 0.800 T. When the plane of the rotor is perpendicular to the direction of the magnetic field, the rotor carries a current of 10.6 mA. In this orientation, the magnetic moment of the rotor is directed opposite the magnetic field. The rotor then turns through one-half revolution....
ADVERTISEMENT
ADVERTISEMENT
ADVERTISEMENT